Prove coordinate vectors are unique for given basis

Click For Summary
The discussion focuses on proving the uniqueness of coordinate vectors in a vector space with respect to a given basis. It suggests starting by assuming that a vector can be expressed as two different linear combinations of the basis vectors with distinct coefficients. The key to the proof lies in utilizing fundamental properties of basis vectors to demonstrate that this assumption leads to a contradiction. By manipulating the equations derived from these linear combinations, one can show that the coefficients must indeed be the same. Ultimately, this establishes that the coordinates of a vector relative to a specific basis are unique.
csc2iffy
Messages
74
Reaction score
0

Homework Statement


Prove that the coordinates of a vector v in a vector space Vn are unique with respect to a given basis B={b1,b2,...,bn}


Homework Equations





The Attempt at a Solution


not sure at all what to do with this
 
Physics news on Phys.org
OK, this is very straightforward. Assume you can represent a vector v as two linear combinations of the basis vectors with different coefficients. After that you only need to use a fundamental property of the basis vectors and that's it.
 
Well you want to show that it's unique. A strategy that's good for this kind of proof is to assume the opposite, work with that assumption, and then arrive at something that shows that it must be unique.

To get started:
Well what's the opposite of being unique? How about we assume that a vector can be represented by this basis in two different ways as a linear combination but with different constants. What do you think you can do with this? Think about how you could manipulate this to show that the constants must be the same. You'll need to know properties of a basis to make it work.
 
Question: A clock's minute hand has length 4 and its hour hand has length 3. What is the distance between the tips at the moment when it is increasing most rapidly?(Putnam Exam Question) Answer: Making assumption that both the hands moves at constant angular velocities, the answer is ## \sqrt{7} .## But don't you think this assumption is somewhat doubtful and wrong?

Similar threads

  • · Replies 6 ·
Replies
6
Views
3K
  • · Replies 4 ·
Replies
4
Views
3K
  • · Replies 1 ·
Replies
1
Views
2K
  • · Replies 4 ·
Replies
4
Views
2K
Replies
15
Views
2K
  • · Replies 1 ·
Replies
1
Views
2K
Replies
4
Views
2K
Replies
3
Views
2K
  • · Replies 9 ·
Replies
9
Views
3K
  • · Replies 9 ·
Replies
9
Views
4K